9 svar
437 visningar
AlvinB är nöjd med hjälpen
AlvinB 4014
Postad: 5 sep 2019 22:33 Redigerad: 25 apr 2022 11:04

Beräkna integral

Hej!

Här följer en kluring bestående av en lite lättare a)-uppgift och en svårare b)-uppgift. Även om uppgifterna är ganska olika är tanken att den som kan lösa a)-uppgiften skall i alla fall kunna komma en liten bit på vägen till att lösa b)-uppgiften. Uppgiften lyder:

Bestäm ett exakt uttryck för värdet på integralen

a) -π2π2cos(x)ex+1 dx\displaystyle\int_{-\frac{\pi}{2}}^{\frac{\pi}{2}}\frac{\cos(x)}{e^x+1}\ dx

b) -11ln(x2)ln(1-x2)ex+1 dx\displaystyle\int_{-1}^1\frac{\ln(x^2)\ln(1-x^2)}{e^x+1}\ dx

Qetsiyah 6503 – Livehjälpare
Postad: 5 sep 2019 22:45 Redigerad: 5 sep 2019 22:47

Den här frågan printar jag ut och skickar till mig själv 10 år fram i tiden. Han kanske kan.

Nej men ärligt talat är det enda jag lärt mig partialbråksuppdelning, variabelsubs och partiell integration. Dessa tar mig ingenstans

AlvinB 4014
Postad: 5 sep 2019 22:55
Qetsiyah skrev:

Den här frågan printar jag ut och skickar till mig själv 10 år fram i tiden. Han kanske kan.

Nej men ärligt talat är det enda jag lärt mig partialbråksuppdelning, variabelsubs och partiell integration. Dessa tar mig ingenstans

Om du kan dessa tre metoder skulle jag säga att du kan lösa a)-uppgiften redan idag. Det är nämligen inget särskilt avancerat som krävs, man måste bara tillämpa metoderna på ett väldigt finurligt sätt. Jag skulle tippa på att du kommer förstå lösningen till a)-uppgiften när du ser den, men antagligen kommer tänka "hur fan kom någon på det här?". :-)

(b)-uppgiften kanske dock ska vänta till en framtida version med lite mer universitetsmatte under bältet.)

Qetsiyah 6503 – Livehjälpare
Postad: 6 sep 2019 19:58 Redigerad: 6 sep 2019 19:59

Då ska jag försöka.

Vad för universitetsmatte?

tomast80 4212
Postad: 6 sep 2019 21:41 Redigerad: 6 sep 2019 21:45

Löste a) enligt nedan:

Visa spoiler

tomast80 skrev:

Löste a) enligt nedan:

Visa spoiler

Så elegant! Får verkligen hålla med omvad AlvinB skrev:

Jag skulle tippa på att du kommer förstå lösningen till a)-uppgiften när du ser den, men antagligen kommer tänka "hur fan kom någon på det här?". :-)

Kan du förklara hur du kom på det? Det är verkligen att krångla till något för att kunna förenkla det sedan!

tomast80 4212
Postad: 7 sep 2019 09:40 Redigerad: 7 sep 2019 09:43

Tackar!

Insåg att täljaren är en jämn funktion och sen är det ett symmetriskt intervall. Det innebär att om man byter ut xx mot -x-x i integralen måste det bli samma värde på hela integralen, d.v.s.:

-π/2π/2cosxex+1dx=\displaystyle \int_{-\pi/2}^{\pi/2}\frac{\cos x}{e^x+1}dx=

-π/2π/2cos(-x)e-x+1dx=\displaystyle \int_{-\pi/2}^{\pi/2}\frac{\cos (-x)}{e^{-x}+1}dx=

{cos(-x)=cosx\cos (-x)=\cos x}

-π/2π/2cosxe-x+1dx\displaystyle \int_{-\pi/2}^{\pi/2} \frac{\cos x}{e^{-x}+1}dx

Därmed kunde jag ta hälften av de två integralerna och lägga ihop och hoppades att delarna med ”e” skulle försvinna, vilket de mycket riktigt gjorde. Det var på ren intuition.

AlvinB 4014
Postad: 7 sep 2019 09:42
tomast80 skrev:

Löste a) enligt nedan:

Visa spoiler

Snyggt, tomast!

Som du konstaterat ger substitutionen t=-xt=-x att:

I=-π2π2cos(x)ex+1 dx=-π2-π2cos(-t)e-t+1 dt=-π2π2cos(t)e-t+1 dtI=\displaystyle\int_{-\frac{\pi}{2}}^\frac{\pi}{2}\frac{\cos(x)}{e^x+1}\ dx=-\int_{\frac{\pi}{2}}^{-\frac{\pi}{2}}\frac{\cos(-t)}{e^{-t}+1}\ dt=\int_{-\frac{\pi}{2}}^\frac{\pi}{2}\frac{\cos(t)}{e^{-t}+1}\ dt

Om vi sedan adderar ihop de två varianterna av integralen får vi en mycket trevlig förenkling:

I+I=-π2π2cos(x)ex+1+cos(x)e-x+1 dx=-π2π2cos(x)ex+1+cos(x)e-x(1+ex) dx\displaystyle I+I=\int_{-\frac{\pi}{2}}^{\frac{\pi}{2}}\frac{\cos(x)}{e^x+1}+\frac{\cos(x)}{e^{-x}+1}\ dx=\int_{-\frac{\pi}{2}}^{\frac{\pi}{2}}\frac{\cos(x)}{e^x+1}+\frac{\cos(x)}{e^{-x}(1+e^x)}\ dx

I=12-π2π2cos(x)ex+1+excosx1+ex dx=12-π2π2cosx(1+ex)ex+1 dx=12-π2π2cosx dxI=\displaystyle\frac{1}{2}\int_{-\frac{\pi}{2}}^{\frac{\pi}{2}}\frac{\cos(x)}{e^x+1}+\frac{e^x\cos\left(x\right)}{1+e^x}\ dx=\frac{1}{2}\int_{-\frac{\pi}{2}}^\frac{\pi}{2}\frac{\cos\left(x\right)(1+e^x)}{e^x+1}\ dx=\frac{1}{2}\int_{-\frac{\pi}{2}}^\frac{\pi}{2}\cos\left(x\right)\ dx

Man kan då börja undra vad som gjorde denna förenkling möjlig. Det är nämligen att cos(x)\cos(x) är en jämn funktion. Vi kan på samma sätt härleda att följande gäller för alla jämna funktioner f(x)f(x):

-aaf(x)ex+1 dx=12-aafx dx=0afx dx\displaystyle\int_{-a}^a\frac{f(x)}{e^x+1}\ dx=\frac{1}{2}\int_{-a}^a f\left(x\right)\ dx=\int_0^a f\left(x\right)\ dx

Detta ger oss även en start på b)-uppgiften, eftersom även dess täljare är jämn. Dock är den resulterande integralen inte lika enkel; den kräver ett par kluriga knep och ganska goda kunskaper om serier och taylorutveckling.

Men vad är det då som gör den där nämnaren, ex+1e^x+1, så speciell? Finns det kanske fler nämnare som ger samma effekt?

Svaret är ja. Om vi söker funktioner g(x)g(x) sådana att:

f(x)g(x)+1+f(x)g(-x)+1=fx\dfrac{f(x)}{g(x)+1}+\dfrac{f(x)}{g(-x)+1}=f\left(x\right)

finner vi efter lite algebra att villkoret

g-x=1g(x)g\left(-x\right)=\dfrac{1}{g(x)}

måste gälla på funktionen g(x)g(x). Vi ser tydligt att g(x)=exg(x)=e^x uppfyller detta eftersom e-x=1/exe^{-x}=1/e^x. Men vi kan även notera att funktioner på formen g(x)=eh(x)g(x)=e^{h(x)} där h(x)h(x) är en udda funktion eftersom det då gäller att eh(-x)=e-h(x)=1/eh(x)e^{h(-x)}=e^{-h(x)}=1/e^{h(x)}. Vi kan till exempel ta h(x)=sin(x)h(x)=\sin(x) och inse att värdena för integralerna:

-π2π2cos(x)esin(x)+1 dx\displaystyle\int_{-\frac{\pi}{2}}^\frac{\pi}{2}\frac{\cos(x)}{e^{\sin(x)}+1}\ dx

-11ln(x2)ln(1-x2)esin(x)+1 dx\displaystyle\int_{-1}^1\frac{\ln(x^2)\ln(1-x^2)}{e^{\sin(x)}+1}\ dx

är samma som i a)a)- respektive b)b)-uppgiften. (Den övre integralen går dock i det här fallet att beräkna med hjälp av substitution)

Men det är inte bara exponentialfunktioner som uppfyller detta villkor. Vi kan även exempelvis ta funktionen:

gx=1-x1+xg\left(x\right)=\dfrac{1-x}{1+x}

som även den uppfyller vårt villkor:

g-x=1+x1-x=11-x1+xg\left(-x\right)=\dfrac{1+x}{1-x}=\dfrac{1}{\frac{1-x}{1+x}}

Man kan även generalisera detta ytterligare och finna att alla funktioner på formen:

gx=p(x)+q(x)p(x)-q(x)g\left(x\right)=\dfrac{p(x)+q(x)}{p(x)-q(x)}

där p(x)p(x) är jämn och q(x)q(x) är udda uppfyller funktionalvillkoret.

En annan fråga är då om det finns något motsvarande villkor för g(x)g(x) om f(x)f(x) är udda. Dessvärre har jag inte lyckats finna några lösningar till funktionalekvationen som då uppstår.

tomast80 4212
Postad: 8 sep 2019 21:41

AlvinB, verkade inte vara någon som lyckades lösa b), så kanske lika bra att du presenterar din lösning?

AlvinB 4014
Postad: 10 sep 2019 20:44 Redigerad: 10 sep 2019 20:45

Ja, mitt mål var att hitta en krånglig integral där integranden är en jämn funktion, men det kanske blev lite väl svårt. Hur som helst följer min lösning här:

Visa spoiler

Samma trick som redan diskuterats ger att:

I=-11ln(x2)ln(1-x2)ex+1 dx=01lnx2ln1-x2 dxI=\displaystyle\int_{-1}^1\frac{\ln(x^2)\ln(1-x^2)}{e^x+1}\ dx=\int_0^1\ln\left(x^2\right)\ln\left(1-x^2\right)\ dx

Maclaurinutvecklingen för ln(1+x)\ln(1+x) gör det sedan möjligt att utveckla ln(1-x2)\ln(1-x^2):

ln1+x=n=1(-1)n+1nxn\displaystyle\ln\left(1+x\right)=\sum_{n=1}^\infty\frac{(-1)^{n+1}}{n}x^n

ln1-x2=n=1(-1)n+1n-x2n=-n=1x2nn\displaystyle\ln\left(1-x^2\right)=\sum_{n=1}^\infty\frac{(-1)^{n+1}}{n}\left(-x^2\right)^n=-\sum_{n=1}^\infty\frac{x^{2n}}{n}

Vår integral blir då:

-01lnx2n=1x2nn dx=-201n=1ln(x)·x2nn dx=\displaystyle-\int_0^1\ln\left(x^2\right)\sum_{n=1}^\infty\frac{x^{2n}}{n}\ dx=-2\int_0^1\sum_{n=1}^\infty\frac{\ln(x)\cdot x^{2n}}{n}\ dx=

Satsen om monoton konvergens låter oss sedan byta plats på summa- och integraltecknet:

=-2n=11n01lnx·x2n dx\displaystyle=-2\sum_{n=1}^\infty\frac{1}{n}\int_0^1 \ln\left(x\right)\cdot x^{2n}\ dx

Integralen kan lösas med partialintegration:

01lnx·x2n dx=[ln(x)·x2n+12n+1]01-01x2n+12n+1·1x=-12n+101x2n dx=0-12n+1[x2n+12n+1]01=-1(2n+1)2\displaystyle\int_0^1\ln\left(x\right)\cdot x^{2n}\ dx=[\frac{\ln(x)\cdot x^{2n+1}}{2n+1}]_0^1-\int_0^1 \frac{x^{2n+1}}{2n+1}\cdot \frac{1}{x}=-\frac{1}{2n+1}\int_0^1 x^{2n}\ dx=0-\frac{1}{2n+1}[\frac{x^{2n+1}}{2n+1}]_0^1=-\frac{1}{(2n+1)^2}

Detta ger:

I=n=12n(2n+1)2\displaystyle I=\sum_{n=1}^\infty\frac{2}{n(2n+1)^2}

Nu har vi omvandlat problemet till att bestämma ovanstående summa. Det är dock inte helt enkelt. Till att börja med kan uttrycket partialbråksuppdelas:

2n(2n+1)2=An+B2n+1+C(2n+1)2\dfrac{2}{n(2n+1)^2}=\dfrac{A}{n}+\dfrac{B}{2n+1}+\dfrac{C}{(2n+1)^2}

där vi finner att A=2A=2B=-4B=-4 och C=-4C=-4. Detta ger:

I=n=12n-42n+1-4(2n+1)2=n=12n-42n+1Σ1-n=14(2n+1)2Σ2=Σ1-Σ2\displaystyle I=\sum_{n=1}^\infty\frac{2}{n}-\frac{4}{2n+1}-\frac{4}{(2n+1)^2}=\underbrace{\sum_{n=1}^\infty\left(\frac{2}{n}-\frac{4}{2n+1}\right)}_{\Sigma_1}-\underbrace{\sum_{n=1}^\infty\frac{4}{(2n+1)^2}}_{\Sigma_2}=\Sigma_1-\Sigma_2

(Uppdelningen rättfärdigas av att Σ1\Sigma_1 och Σ2\Sigma_2 är absolutkonvergenta)

Σ1=n=12n-42n+1=2n=11n-22n+1=21-23+12-25+...=222-23+24-25+...=\displaystyle\Sigma_{1}=\sum_{n=1}^\infty\frac{2}{n}-\frac{4}{2n+1}=2\sum_{n=1}^\infty\frac{1}{n}-\frac{2}{2n+1}=2\left(1-\frac{2}{3}+\frac{1}{2}-\frac{2}{5}+...\right)=2\left(\frac{2}{2}-\frac{2}{3}+\frac{2}{4}-\frac{2}{5}+...\right)=

=412-13+14-15+...=4n=2(-1)nn\displaystyle=4\left(\dfrac{1}{2}-\frac{1}{3}+\frac{1}{4}-\frac{1}{5}+...\right)=4\sum_{n=2}^\infty\frac{(-1)^n}{n}

Denna summa kan bestämmas med hjälp av den ovannämnda maclaurinutvecklingen för ln(1+x)\ln(1+x):

ln1+x=n=1(-1)n+1n·xn\displaystyle \ln\left(1+x\right)=\sum_{n=1}^\infty\frac{(-1)^{n+1}}{n}\cdot x^n

ln2=n=1(-1)n+1n\displaystyle\ln\left(2\right)=\sum_{n=1}^\infty\frac{(-1)^{n+1}}{n}

-ln2=n=1(-1)nn\displaystyle-\ln\left(2\right)=\sum_{n=1}^\infty\frac{(-1)^n}{n}

1-ln2=n=2(-1)nn\displaystyle1-\ln\left(2\right)=\sum_{n=2}^\infty\frac{(-1)^n}{n}

Alltså är

Σ1=4(1-ln(2))=4-4ln(2)\Sigma_1=4(1-\ln(2))=4-4\ln(2)

Med hjälp av den kända summan

π26=n=11n2\displaystyle\frac{\pi^2}{6}=\sum_{n=1}^\infty\frac{1}{n^2}

kan Σ2\Sigma_2 bestämmas:

π224=n=114n2=n=11(2n)2\displaystyle\frac{\pi^2}{24}=\sum_{n=1}^\infty\frac{1}{4n^2}=\sum_{n=1}^\infty\frac{1}{(2n)^2}

π26-π224=n=11n2-n=11(2n)2=1+122+132+...-122-142-...=1+132+152+...\displaystyle\frac{\pi^2}{6}-\frac{\pi^2}{24}=\sum_{n=1}^\infty\frac{1}{n^2}-\sum_{n=1}^\infty\frac{1}{(2n)^2}=1+\frac{1}{2^2}+\frac{1}{3^2}+...-\frac{1}{2^2}-\frac{1}{4^2}-...=1+\frac{1}{3^2}+\frac{1}{5^2}+...

π28=1+132+152+...=1+n=11(2n+1)2\displaystyle\frac{\pi^2}{8}=1+\frac{1}{3^2}+\frac{1}{5^2}+...=1+\sum_{n=1}^\infty\frac{1}{(2n+1)^2}

π28-1=n=11(2n+1)2\displaystyle\frac{\pi^2}{8}-1=\sum_{n=1}^\infty\frac{1}{(2n+1)^2}

Alltså blir Σ2\Sigma_2:

Σ2=4(π28-1)=π22-4\Sigma_2=4(\dfrac{\pi^2}{8}-1)=\dfrac{\pi^2}{2}-4

Integralen blir då:

I=Σ1-Σ2=4-4ln2-(π22-4)=8-4ln2-π22I=\Sigma_1-\Sigma_2=4-4\ln\left(2\right)-(\dfrac{\pi^2}{2}-4)=8-4\ln\left(2\right)-\dfrac{\pi^2}{2}

Och slutligen får vi då:

-11ln(x2)ln(1-x2)ex+1 dx=8-4ln2-π22\displaystyle\boxed{\int_{-1}^1\frac{\ln(x^2)\ln(1-x^2)}{e^x+1}\ dx=8-4\ln\left(2\right)-\frac{\pi^2}{2}}

Svara Avbryt
Close